2017 제78회 William Lowell Putnam 수학경시대회

2017년 12월 2일. 출처

GD Star Rating
loading...
2017 제78회 William Lowell Putnam 수학경시대회, 4.5 out of 5 based on 2 ratings

집합 $S$는 아래 세 조건을 만족시키는 양의 정수들의 집합 중 가장 작은 것이다.

(a) $2$는 $S$의 원소이다.

(b) $n^2$이 $S$의 원소이면 $n$ 또한 $S$의 원소이다.

(c) $n$이 $S$의 원소이면, $(n+5)^2$ 또한 $S$의 원소이다.

$S$에 속하지 않은 양의 정수들은?

(집합 $S$가 가장 작다는 뜻은 $S$가 그러한 성질을 갖는 다른 모든 집합의 부분집합이라는 뜻이다.)

다항식 $Q_0(x) = 1$, $Q_1(x) = x$이며 정수 $n\ge 2$에 대하여  \[Q_n(x) = \frac{(Q_{n-1}(x))^2 – 1}{Q_{n-2}(x)}\]이라고 하자. 모든 양의 정수 $n$에 대하여 $Q_n(x)$는 정수 계수 다항식임을 보여라.

두 실수 $a<b$가 있다. 두 연속함수 $f:[a,b]\to(0,\infty)$가 $\int_a^b f(x)\,dx = \int_a^b g(x)\,dx$이지만, $f \neq g$이라고 한다. 각각의 양의 정수 $n$에 대하여 \[I_n = \int_a^b \frac{(f(x))^{n+1}}{(g(x))^n}\,dx\]이라고 정의하자. 이때 $I_1, I_2, I_3, \dots$는 증가수열이며 $\lim_{n \to \infty} I_n = \infty$임을 보여라.

총 $2N$명의 학생이 있는 학급에서 퀴즈 시험을 보았으며, 가능한 점수는 $0$, $1$, $\ldots$, $10$점 중 하나였다. 각각의 점수는 적어도 한 번 이상 나왔고, 평균은 정확히 $7.4$였다. 이때 이 학급을 학생 $N$명씩 두 그룹으로 잘 나누어서 각 그룹의 학생들의 성적 평균이 정확히 $7.4$가 되게 할 수 있음을 보여라.

정수 $1$부터 $n$까지 적힌 $n$장의 카드가 있다. 이 카드를 잘 섞어서 수가 보이지 않게 쌓아두었다. 세 사람 $A$, $B$, $C$가 $A$부터 시작하여 $A$, $B$, $C$, $A$, $\ldots$ 순으로 돌아가며 카드 중에 하나를 임의로 뽑는다. (이때 남아있는 카드 중 각 카드를 뽑을 확률은 동일하다.) 카드를 하나 뽑으면 그 카드와 함께 그 카드에 적힌 수보다 큰 수를 가진 카드를 모두 빼서 버리고 남은 카드는 다시 잘 섞어둔다. 정수 $1$이 적힌 카드를 뽑는 사람이 나올때까지 게임을 계속하며 $1$을 뽑은 사람이 게임을 이긴다.

세 사람 각각에 대하여, 그 사람이 이길 확률이 가장 높게 될 $n$이 있으며 그러한 $n$이 얼마든지 커질 수 있음을 보여라.

정이십면체의 30개 변 각각에 $1$, $2$, $3$, $\ldots$, $30$의 번호를 붙여서 구분하기로 하였다. 각 변을 빨강, 흰색, 파랑색 중 하나로 칠하되, 삼각형 모양의 면 $20$개 모두 두 변은 색이 같고 다른 한 변은 색이 다르게 칠하고자 한다. 이렇게 칠하는 방법의 수는 얼마인가?

평면 위에 서로 다른 직선 $L_1$, $L_2$가 있다. 이 두 직선 $L_1$ and $L_2$가 만날 필요충분조건은 모든 실수 $\lambda\neq 0$와 $L_1$, $L_2$ 위에 있지 않은 임의의 점 $P$에 대하여,  $\overrightarrow{PA_2} = \lambda \overrightarrow{PA_1}$이 되는 $L_1$ 위의 점 $A_1$, $L_2$ 위의 점 $A_2$가 존재하는 것임을 보여라.

양의 정수 $N$이 \[N = a + (a+1) +(a+2) + \cdots + (a+k-1)\]과 같이 $k$개의 연속한 양의 정수의 합으로 표현될 수 있는 $k>1$가 $k=2017$인 경우 밖에 없다고 한다. 이러한 성질을 갖는 양의 정수 $N$ 중에서 $a$ 값이 가장 작은 것은 무엇이겠는가?

멱급수 $f(x) = \sum_{i=0}^\infty c_i x^I$의 각각의 계수 $c_i$가 $0$ 또는 $1$이라고 한다. 만일 $f(2/3) = 3/2$이면, $f(1/2)$는 무리수일 수 밖에 없음을 보여라.

다음 식의 값을 구하여라.\begin{gather*}\sum_{k=0}^\infty \left( 3 \cdot \frac{\ln(4k+2)}{4k+2} – \frac{\ln(4k+3)}{4k+3} – \frac{\ln(4k+4)}{4k+4} – \frac{\ln(4k+5)}{4k+5} \right) \\= 3 \cdot \frac{\ln 2}{2} – \frac{\ln 3}{3} – \frac{\ln 4}{4} – \frac{\ln 5}{5}+ 3 \cdot \frac{\ln 6}{6} – \frac{\ln 7}{7} \\ – \frac{\ln 8}{8} – \frac{\ln 9}{9}+ 3 \cdot \frac{\ln 10}{10} – \cdots .\end{gather*} (단, $\ln x$는 $x$의 자연로그값을 뜻한다.)

어떤 삼각형 $T$와 같은 평면 위에 있는 직선이 이 삼각형을 두 영역으로 나누었을 때 이 두 영역의 넓이도 같고 둘레도 같다면 이 직선을 삼각형 $T$의 이등분선이라고 부르자. 세 변의 길이가 각각 $a$, $b$, $c$인 삼각형이 정확히 두 개의 이등분선을 가진다고 할 때 가능한 세 양의 정수 $a>b>c$ 중에서 $a$ 값이 가장 작을 때 $a$, $b$, $c$를 구하라.

집합 $\{1,2,\ldots,2017\}$의 서로 다른 원소인 $x_0$, $x_1$, $\ldots$, $x_{63}$의 순서쌍 $(x_0,x_1,\ldots,x_{63})$ 중에서 \[ x_0+x_1+2x_2+3x_3+\cdots+63x_{63}\]이 $2017$의 배수가 되는 것의 개수를 구하여라.

답글 남기기